2021 Fall AMC 10A Problems/Problem 14

Revision as of 21:18, 23 November 2021 by Ihatemath123 (talk | contribs)

Problem

How many ordered pairs $(x,y)$ of real numbers satisfy the following system of equations? \[x^2+3y=9\] \[(|x|+|y|-4)^2 = 1\]

$\textbf{(A )} 1 \qquad\textbf{(B) } 2 \qquad\textbf{(C) } 3 \qquad\textbf{(D) } 5 \qquad\textbf{(E) } 7$


Solution (Graphing)

The second equation is $(|x|+|y| - 4)^2 = 1$. We know that the graph of $|x| + |y|$ is a very simple diamond shape, so let's see if we can reduce this equation to that form.

$(|x|+|y| - 4)^2 = 1 \implies |x|+|y| - 4 = \pm 1 \implies |x|+|y| = \{3,5$}.

We now have two separate graphs for this equation and one graph for the first equation, so let's put it on the coordinate plane:


[asy]  Label f;  f.p=fontsize(6);  xaxis(-8,8,Ticks(f, 1.0,0.5));  yaxis(-8,8,Ticks(f, 1.0,0.5));  real f(real x)  {  return 3-x;  }  draw(graph(f,-2,2));   [/asy]

Solution 2 (Unrigorous but pretty standard)

We can manipulate the first equation to get $y = -\frac{x^{2}}{3} + 3$. From the second equation, we have that $|x|+|y|-4 = 1$ or $|x|+|y|-4 = -1$. We will consider each case separately.

If $|x|+|y|-4 = 1$, then $|x|+|y| = 5$. The graph of this is a square with vertices $(5,0)$, $(-5,0)$, $(0,5)$ and $(0,-5)$. The parabola from the first equation is downwards facing, and its vertex is inside this square; the parabola will clearly intersect the square twice. Therefore, this case gives us $\underline{2}$ solutions.

If $|x|+|y|-4 = -1$, then $|x|+|y| = 3$. The graph of this is a square with vertices $(3,0)$, $(-3,0)$, $(0,3)$ and $(0,-3)$. The vertex of the parabola from the first equation is on one of the corners of this square (in particular, $(0,3)$). Also, at $y = 0$, the parabola has $x$ intercepts of $\pm 3$; the square passes through both of those points. If we continue to move down, the square narrows in, while the parabola continues to expand. Therefore, these are our only $3$ intersection points in this case: $(0,3)$, $(3,0)$ and $(-3,0)$. This case gives us $\underline{3}$ solutions.

Adding these two cases together, we get our final answer of $\boxed{\textbf{(D) } 5}$.

See Also

2021 Fall AMC 10A (ProblemsAnswer KeyResources)
Preceded by
Problem 13
Followed by
Problem 15
1 2 3 4 5 6 7 8 9 10 11 12 13 14 15 16 17 18 19 20 21 22 23 24 25
All AMC 10 Problems and Solutions

The problems on this page are copyrighted by the Mathematical Association of America's American Mathematics Competitions. AMC logo.png